1

Let's consider a liquid that contains either dextrogyre or levogyre molecules, but only a highly preponderance of one of them.

Since the molecules are randomly oriented in the liquid, how is it possible to have a net rotation power of the liquid since the various angular orientations of the molecules should cancel out the effect of the light rotation ?

  • Assuming you mean "only one sort" (and even a preponderance of one) you might look here https://chemistry.stackexchange.com/q/6271/48509 and here https://physics.stackexchange.com/questions/15503/how-do-optically-active-compounds-rotate-plane-polarized-light – Alchimista Aug 13 '20 at 09:43
  • @Alchimista : thank you very much. Yes I was meaning only one sort of them (I'm not fluent in english). Thank you. I will look at your very useful links. – Mathieu Krisztian Aug 13 '20 at 10:03

0 Answers0